Đến nội dung

KietLW9 nội dung

Có 1000 mục bởi KietLW9 (Tìm giới hạn từ 30-03-2020)



Sắp theo                Sắp xếp  

#741312 Tìm tất cả số nguyên $n$ thỏa mãn $n(n+1)...(n+p-3)\equ...

Đã gửi bởi KietLW9 on 05-09-2023 - 21:57 trong Số học

?

Sao vậy bạn, câu a khá cơ bản còn câu b chỉ cần dùng hensel là được mà




#741301 Tìm tất cả số nguyên $n$ thỏa mãn $n(n+1)...(n+p-3)\equ...

Đã gửi bởi KietLW9 on 04-09-2023 - 18:37 trong Số học

Dùng Hensel nhé




#739726 ĐỀ THI CHỌN HỌC SINH GIỎI CẤP HUYỆN TOÁN 8 PHÒNG GD&ĐT HIỆP HÒA 2020-2021

Đã gửi bởi KietLW9 on 31-05-2023 - 00:12 trong Tài liệu - Đề thi

Xét dãy số $(x_n)$ xác định bởi $x_0=0,x_1=1$ và $$x_{n+2}=2x_{n+1}+2x_n,\forall n \in \mathbb{N}$$

Chứng minh rằng $v_3(n)=v_3(a_n)$




#739072 $\sum \frac{a}{\sqrt{a^{2}+...

Đã gửi bởi KietLW9 on 07-05-2023 - 00:09 trong Bất đẳng thức và cực trị

$$x^p+y^p=(x+y)^z$$

Xét $p=2$ thì $z=1$ nên $x=y=1$, vậy bộ nghiệm $(a,b,c,p)=(1,1,1,2)$ thỏa mãn

 

Xét $p$ là snt lẻ, giả sử $x+y$ có ước nguyên tố lẻ $r \neq p$, khi đó $x,y$ không thể đồng thời bằng $1$ và $v_r(x+y)=zv_r(x+y)\Rightarrow z=1\Rightarrow x^p+y^p=x+y$, mâu thuẫn

Do vậy nếu $x+y$ có ước nguyên tố lẻ đó là $p$, tức là $$v_p(x+y)+1=zv_p(x+y)$$ nên $z=2$, khi đó $x^3+y^3\leq x^p+y^p\leq 2(x^2+y^2)\Rightarrow x=2,y=1\Rightarrow p=3$, do đó có thêm một bộ là $(x,y,z,p)=(2,1,2,3)$ hoặc ngược lại vs $x,y$




#736364 Chứng minh rằng: Đường tròn đường kính $ST$ trực giao với đường trò...

Đã gửi bởi KietLW9 on 20-12-2022 - 12:08 trong Hình học

Taiwan TST 2017




#735893 Tìm trong tập hợp đó một đa thức có bậc bé nhất nhưng có nghiệm lớn nhất

Đã gửi bởi KietLW9 on 25-11-2022 - 22:15 trong Đa thức

Giả sử $a$ là một nghiệm của $P(x)$ thì do $P(a^2-1)=P(a)P(-a)=0$ nên $a^2-1$ cũng là một nghiệm của $P(x)$. 

Từ đó nếu $a>\frac{1+\sqrt{5}}{2}$ thì ta có thể biễu diễn nghiệm của $P(x)$ bởi một dãy tăng vô hạn các số. Vô lí vì $P(x)$ là một đa thức khác không. Do vậy $a\leq \frac{1+\sqrt{5}}{2}$

Đa thức $P(x)=x^2-x-1$ có nghiệm $\frac{1+\sqrt{5}}{2}$ và đa thức này thỏa mãn đề bài nên ta cần chỉ ra không tồn tại đa thức bậc nhất thỏa mãn

Thật vậy, giả sử có $P(x) =ax+b$ trong đó $a$ khác $0$ thì thay vào đề bài ta được $$ax^2+(b-a)=-a^2x^2+b^2$$

Do đó $a=-1$ và $b=\frac{1\pm \sqrt{5}}{2}$

Nhưng trong trường hợp này cả hai đa thức đều có nghiệm nhỏ hơn $\frac{1+\sqrt{5}}{2}$

 




#735787 CÁC BÀI TOÁN VỀ ĐA THỨC

Đã gửi bởi KietLW9 on 20-11-2022 - 10:31 trong Đa thức

$\textbf{Bài toán 7.}$ Cho $n\geq 2$ là một số nguyên. Chứng minh rằng đa thức $$P(x)=\sum_{k=0}^{n}\frac{x^k}{k!}$$ không có nghiệm hữu tỉ.

$\textbf{Lời giải.}$ 

Ta thấy rằng $P(x)$ có nghiệm hữu tỉ là $\alpha $ thì $\alpha $ cũng là nghiệm của phương trình $$x^n+nx^{n-1}+...+\frac{n!}{1!}x+n!=0$$

Suy ra $\alpha ^n+n\alpha ^{n-1}+...+\frac{n!}{1!}\alpha +n!=0$

Gọi $p$ là một ước nguyên tố bất kì của $n$

Với $k=\overline{1,n}$ ta đặt $v_p(k!)=r_k$ thì $r_k=\left [ \frac{k}{p} \right ]+\left [ \frac{k}{p^2} \right ]+...+\left [ \frac{k}{p^s} \right ]$ trong đó $s$ là số thỏa mãn $p^s\leq k<p^{s+1}$

$\Rightarrow r_k\leqslant p(\frac{1}{p}+\frac{1}{p^2}+...+\frac{1}{p^s})=k.\frac{p^s-1}{p^s}<k$ nên $r_n-r_k>r_n-k\Rightarrow r_n-r_k\geqslant r_n-k+1\Rightarrow p^{r_n-k+1}|\frac{n!}{k!}$

Vì $p|n$ nên $p|\alpha$ do đó $p^k|\alpha^k$ suy ra $p^{r_n+1}|\frac{n!}{k!}\alpha ^k,\forall k = \overline{1,n}$

Suy ra $p^{r_n+1}|n!$. Vô lí do $v_p(n!)=r_n$

Vậy đa thức $P(x)$ không có nghiệm hữu tỉ.




#735785 CÁC BÀI TOÁN VỀ ĐA THỨC

Đã gửi bởi KietLW9 on 20-11-2022 - 10:01 trong Đa thức

$\textbf{Bài toán 6.}$ Cho $P,Q$ là các đa thức hệ số thực thỏa mãn $$P^2(x)-Q^3(x)=2022$$ Chứng minh rằng $P,Q$ là các đa thức hằng.

$\textbf{Lời giải.}$ Giả sử $\text{deg P = a , deg Q = b }$ sao cho $a,b>0$ thì ta thấy $2a=3b$

Đạo hàm hai vế, ta được: $2P'(x)P(x)=3Q^2(x)Q'(x)$ nên $2P'(x)P(x)$ chia hết cho $Q^2(x)$

Lại có $P^2(x)-2022$ chia hết cho $Q^2(x)$ nên biến đổi đại số ta được: $4044P'(x)$ chia hết cho $Q(x)$

Từ đó ta có $a-1\geq 2b=\frac{4}{3}a$ suy ra mâu thuẫn

Do vậy phải có ít nhất một trong hai số $a,b$ bằng $0$. Tóm lại $P,Q$ là các đa thức hằng.




#735783 CÁC BÀI TOÁN VỀ ĐA THỨC

Đã gửi bởi KietLW9 on 20-11-2022 - 09:41 trong Đa thức

$\textbf{Bài toán 5.}$ Cho hai đa thức hệ số nguyên, monic là $P(x)$ và $Q(x)$ trong đó deg $P=3$ và deg $Q=2$. Giả sử rằng $P(x)$ có ba nghiệm vô tỷ phân biệt  $a,b,c$ sao cho $a+b+c=0$ và $Q(a)=b$. 
a) Chứng minh rằng $P(Q(x))$ chia hết cho $P(x)$
b) Chứng minh rằng $Q(a)+Q(b)+Q(c)=0$.
$\textbf{Lời giải.}$ 
a) Trước tiên ta có một nhận xét sau: Xét một số vô tỉ $\alpha$ và đa thức hệ số nguyên $P(x)$ có bậc nhỏ nhất nhận $\alpha$ làm nghiệm được gọi là đa thức "tối tiểu" ứng với $\alpha$. Và đương nhiên, mọi đa thức nhận $\alpha$ làm nghiệm đều chia hết cho $P(x)$.
Quay lại bài toán ta thấy, không thế có một đa thức bậc nhất nào nhận $a$ là nghiệm nên ta giả sử $f(x)$ là đa thức bậc hai nhận $a$ làm nghiệm. Thực hiện phép chia đa thức ta thấy
$$P(x)=Q(x)f(x)+R(x)$$ trong đó $\text{deg R}<2$ mà $R(a)=0$ nên $R\equiv 0\Rightarrow P(x)=Q(x)f(x)$. Do đó $Q(x)$ là đa thức bậc nhất hệ số hữu tỉ nhận $a$ làm nghiệm, vô lí. 
Vậy $P(x)$ là đa thức "tối tiểu" đối với số $a$
Kết hợp với $P(Q(a))=P(b)=0$ nên suy ra được $P(Q(x))$ chia hết cho $P(x)$
b) Từ câu a) ta thất $Q(b),Q(c)$ cũng là các nghiệm của $P(x)$ nên $Q(b),Q(c) \in \left \{ a;b;c \right \}$
* Nếu $Q(b)=b$ thì $Q(x)-x$ là tam thức bậc hai nhận $b$ làm nghiệm nên vô lí
* Nếu $Q(b)= a$ thì đặt $Q(x)= x^2+px+q$, ta giải hệ $\left\{\begin{matrix}b^2+pb+q=a & \\ a^2+pa+q=b & \end{matrix}\right.\Rightarrow a+b+p+1=0\Rightarrow c=p+1$ (Vô lí vì $c$ là số vô tỉ)
Vậy $Q(b)=c$. Tương tự ta có: $Q(c)=a$ nên $Q(a)+Q(b)+Q(c)=0$.



#735782 CÁC BÀI TOÁN VỀ ĐA THỨC

Đã gửi bởi KietLW9 on 20-11-2022 - 09:24 trong Đa thức

$\textbf{Bài toán 4.}$ Tìm tất cả các đa thức $P(x)$ hệ số thực, khác hằng thỏa mãn $$P(x^2)=P(x)P(x-1),\forall x \in \mathbb{R}$$

$\textbf{Lời giải.}$ Ta thấy rằng nếu $r$ là một nghiệm của đa thức $P(x)$ thì thay $x$ bởi $r+1$ ta suy ra $(r+1)^2$ cũng là một nghiệm của $P(x)$ mà $(r+1)^2>r$ nên dãy nghiệm của $P(x)$ sẽ tăng tới vô cùng nên $P\equiv 0$, vô lí do $P(x)$ khác hằng.

Đồng nhất hệ số cao nhất ta dễ có P(x) monic và từ chứng minh trên ta suy ra $P(x)$ có bậc chẵn

Do đó $P(x)$ sẽ có dạng $$P(x)=(x^2+x+1)^n+Q(x)$$ trong đó $n$ là số nguyên dương

Xét trường hợp $Q\not\equiv 0$ thì $\text{deg Q}=k<2n$

Thay vào phương trình ban đầu ta được: $Q(x^2)=Q(x)(x^2-x+1)^n+Q(x-1)(x^2+x+1)^n+Q(x)Q(x-1)$

Đồng nhất bậc cao nhất ta được $2k=2n+k$ suy ra $k=2n$ (vô lí) do đó $Q\equiv 0$ hay $$P(x)=(x^2+x+1)^n,\forall x \in \mathbb{R}$$




#735779 CÁC BÀI TOÁN VỀ ĐA THỨC

Đã gửi bởi KietLW9 on 20-11-2022 - 09:12 trong Đa thức

$\textbf{Bài toán 3.}$ Cho các số nguyên dương $a,b>1$ sao cho tồn tại hai đa thức $P(x),Q(x)$ hệ số thực thỏa mãn $P(x)>0, \forall x \in \mathbb{R}$ và $$P(Q(x))=(P(x))^a(Q(x))^b,\forall x \in \mathbb{R}$$
Chứng minh rằng tồn tại số thực $m$ sao cho $P'(m)=Q'(m)=0$.
$\textbf{Lời giải. }$
Vì $P(x)>0, \forall x \in \mathbb{R}$ nên $Q(x)>0,\forall x \in \mathbb{R}$ hay nói cách khác $Q(x)$ không có nghiệm thực nên $Q(x)$ có bậc chẵn
Đạo hàm hai vế ta được: $$Q'(x).P'(Q(x))=a.P'(x).(P(x))^{a-1}.(Q(x))^b+b.Q'(x).(Q(x))^{b-1}.(P(x))^a(*)$$
Rõ ràng $Q'(x)$ có bậc lẻ nên tồn tại số thực $m$ để $Q'(m)=0$
Thay $m$ vào $(*)$ thì ta có ngay $a.P'(m).(P(m))^{a-1}.(Q(m))^b=0\Rightarrow P'(m)=0$
Vậy ta chọn được $m$ thỏa mãn $P'(m)=Q'(m)=0$.



#735777 CÁC BÀI TOÁN VỀ ĐA THỨC

Đã gửi bởi KietLW9 on 20-11-2022 - 09:05 trong Đa thức

$\textbf{Bài toán 2 (Thailand MO 2014).}$ Tìm tất cả các đa thức $P(x)$ có hệ số nguyên thỏa mãn $$P(n)|2557^n+213.2014,\forall n \in \mathbb{Z}^+$$

$\textbf{Lời giải.}$ Rõ ràng $P\equiv 1$ và $P\equiv -1$ là hai đa thức thỏa mãn bài toán. Bây giờ ta xét $P(x)$ là đa thức hằng khác $0,-1,1$ hoặc $P(x)$ là đa thức chứa biến thì luôn tồn tại một số nguyên dương $n_0$ thỏa mãn $|P(n_0)|\geq 2$ 

Gọi $p$ là một ước nguyên tố của $P(n_0)$ thì $p|2557^{n_0}+213.2014$

Mặt khác thì $p|P(n_0+p)-P(n_0)|2557^{n_0}(2557^p-1)$. Rõ ràng $p \notin \left \{ 2;3;19;53;71;2557 \right \}$ do $p|2557^{n_0}+213.2014$

Từ đó suy ra $p|2557^p-1$

Theo định lý Fermat ta có được: $p|2557^p-2557\Rightarrow p|2556\Rightarrow p \in \left \{ 2;3;71 \right \}$ nên ta thấy vô lí

Vậy không tồn tại đa thức nào thỏa mãn trong trường hợp này

Vậy $P\equiv 1$ và $P\equiv -1$

 



#735774 CÁC BÀI TOÁN VỀ ĐA THỨC

Đã gửi bởi KietLW9 on 20-11-2022 - 08:44 trong Đa thức

$\textbf{Bài toán 1.}$ Cho $P,Q$ là hai đa thức hệ số nguyên, khác đa thức hằng. Xét dãy số $$a_n=2016^{P(n)}+Q(n),\forall n \geq 1$$ Chứng minh rằng tồn tại vô hạn số nguyên tố $p$ thỏa mãn tính chất: Ứng với mỗi số nguyên tố $p$ đó thì luôn tồn tại số nguyên dương $m$ để $p|x_m$.

$\textbf{Lời giải.}$  Ta có: $x_n=2016^{P(n)}-2016^{P(1)}+Q(n)+2016^{P(1)}=2016^{P(n)}-2016^{P(1)}+R(n)$ trong đó $R(n)=Q(n)+2016^{P(1)}$ là đa thức hệ số nguyên, khác hằng.

Theo định lý Schur thì tồn tại vô số số nguyên tố $p>2016$ thỏa mãn tính chất: Tồn tại số nguyên dương $n$ để $p|R(n)$

Theo định lý thặng dư trung hoa ta chọn được số nguyên dương $m$ sao cho $P(m)>P(1),m>p$ và $m\equiv n(\text{mod p})$ đồng thời $m\equiv 1(\text{mod p-1})$

Từ đây ta thấy $p|m-n|R(m)-R(n)\Rightarrow p|R(m)$

Mặt khác thì $p-1|m-1|P(m)-P(1)$ và do $(2016,p)=1$ nên theo định lý Fermat thì $p|2016^{P(m)}-2016^{P(1)}$

Do vậy luôn tồn tại số nguyên dương $m$ để $p|x_m$ mà ta lại chọn được vô hạn số nguyên tố $p$ như vậy nên có ngay điều phải chứng minh.




#735772 TOPIC [MỘT SỐ BÀI TOÁN SỐ HỌC]

Đã gửi bởi KietLW9 on 20-11-2022 - 01:33 trong Số học

$\textbf{Bài toán 7.}$ Chứng minh rằng với mọi $n$ nguyên dương luôn tồn tại $m$ để $$7^n|3^m+5^m-1$$

$\textbf{Lời giải.}$ Chọn $m=7^{n-1}$ và sử dụng LTE, ta có: $$v_7(8^m-1)=v_7(8-1)+v_7(m)=n\Rightarrow 7^n|8^m-1=(2^m-1)(4^m+2^m+1)$$

Mặt khác $m$ không là bội của $3$ nên $7^n \not| 2^m-1\Rightarrow 7^n|4^m+2^m+1$

Kết hợp với $m$ lẻ ta có: $v_7(4^m+3^m)=v_7(3+4)+v_7(m)=n\Rightarrow 4^m\equiv -3^m$ (mod $7^n$)

Tương tự ta cũng có: $2^m\equiv -5^m$ (mod $7^n$) nên $3^m+5^m-1\equiv 0$ (mod $7^n$)

$\textbf{Bài toán 8.}$ Cho $p$ là một số nguyên tố lẻ. Chứng minh rằng mọi ước nguyên tố của $\frac{p^{2p}+1}{p^2+1}$ đều có dạng $4k+1$

$\textbf{Lời giải.}$ Rõ ràng với $p$ lẻ thì $\frac{p^{2p}+1}{p^2+1}$ lẻ nên tất cả các ước nguyên tố của $\frac{p^{2p}+1}{p^2+1}$ đều lẻ. Gọi $q$ là một trong số chúng $\Rightarrow q|p^{4p}-1$

Gọi $h$ là cấp của $p$ theo modulo $q$ thì $h|4p$ nên $h \in \left \{ 1;2;4;p;2p;4p \right \}$

Nếu $h \in \left \{ 1;2;p;2p \right \}$ thì ta luôn có $p^{2p}\equiv 1(\text{mod q})$, vô lí với $p^{2p}\equiv -1(\text{mod q})$

* Nếu $h=4p$ thì $q\equiv 1(\text{mod 4p})$

* Nếu $h=4$ thì $q$ vẫn có dạng $4k+1$

Do đó ta có điều phải chứng minh

$\textbf{Bài toán 9.}$ Cho $a,m,n$ là các số nguyên lớn hơn $1$. Chứng minh rằng nếu $a^2+a+1|(a+1)^m+a^n$ thì $3|m+n$




#735626 [TOPIC] TỔNG HỢP CÁC BÀI TOÁN VỀ PHƯƠNG TRÌNH HÀM

Đã gửi bởi KietLW9 on 07-11-2022 - 16:49 trong Phương trình hàm

$\textbf{Bài toán 2.}$ Xét một bảng ô vuông vô hạn và đặt một quân mã vào ô bất kì. Quân mã này thay vì đi các $1 \times 2$ hay $2 \times 1$ như bàn cờ vua thông thường thì ở bảng vuông này nó có thể đi theo cách $m \times n$ hoặc $n \times m$ trong đó $m,n$ là các số nguyên dương bất kì cho trước. Chứng minh rằng với mọi $m,n$, sau một số lần di chuyển, quân mã không thể q




#735547 LUYỆN TẬP SỐ HỌC

Đã gửi bởi KietLW9 on 01-11-2022 - 18:07 trong Số học

LUYỆN TẬP SỐ HỌC

$\textbf{Bài toán 1.}$ Chứng minh rằng với mọi $k$ nguyên dương thì $2k^4+4k^3+3k^2+k$ không là tích của hai số nguyên dương liên tiếp.

$\textbf{Bài toán 2.}$ Xét dãy số $(x_n)$ được xác định bởi $x_1=6$ và $$x_{n+1}=x_n+gcd(x_{n},n),\forall n \geq 1$$

Chứng minh rằng $x_{n+1}-x_n$ hoặc bằng $1$ hoặc là số nguyên tố

$\textbf{Bài toán 3.}$ Cho dãy số $(a_n)$ được xác định bởi $a_1=1,a_2=3$ và $$a_{n+2}=2a_{n+1}a_n+1,\forall n \geq 1$$
Tìm số nguyên dương $k$ lớn nhất mà $2^k|a_{2022}-a_{2021}$.

 




#735534 [Số học] THPT tháng 12: Chứng minh tồn tại vô hạn $n$ thoả $n...

Đã gửi bởi KietLW9 on 31-10-2022 - 00:07 trong Thảo luận đề thi VMEO IV

vf




#735533 [Số học] THPT tháng 12: Chứng minh tồn tại vô hạn $n$ thoả $n...

Đã gửi bởi KietLW9 on 31-10-2022 - 00:06 trong Thảo luận đề thi VMEO IV

Dạo này bận quá nên mới vô được diễn đàn  :( Bài này chắc thầy Hà Duy Hưng chế ra từ các tính chất của các số  Fermat

$\textbf{Lời giải.}$

Ta xét các số $F_n=2^{2^n}+1$ trong đó $n$ là một số tự nhiên

Rõ ràng tất cả các số hạng của dãy trên đều nguyên tố cùng nhau và nếu $n\geq 1$ thì $F_n$ sẽ không có ước nguyên tố dạng $4k+3$.

Gọi $p_i$ là một ước nguyên tố của $F_i$ thì $p_0=3$ và $p_1,p_2,...,p_{k-1}$ là các số nguyên tố dạng $4t+1$ và các số này đều phân biệt

Ta chọn $n_{k-1}=p_0p_1...p_{k-1}$ thì $n_{k-1}$ sẽ có đúng $k$ ước nguyên tố phân biệt, và ngoại trừ số $3$ ra thì mọi ước nguyên tố của $n_{k-1}$ đều có dạng $4t+1$. Ta chứng minh đây là số thỏa mãn đề bài bằng cách chỉ ra $n_{k-1}|2^{\sigma (n_{k-1})}-1$

Thậy vậy, dễ dàng có $2^k| \sigma (n_{k-1})$ nên ta cần chứng minh: $n_{k-1}|2^{2^k}-1$

Mà mỗi ước nguyên tố của $n_{k-1}$ đều đôi một phân biệt nên bài toán sẽ hoàn tất nếu ta chỉ ra $p_i|2^{2^k}-1,\forall i = \overline{0,k-1}$

Đây là điểu hiển nhiên do $F_i|2^{2^k}-1,\forall i =\overline{0,k-1}$

 




#735473 Với mỗi số nguyên dương $k$, ta định nghĩa $S(k)$ là tổng...

Đã gửi bởi KietLW9 on 26-10-2022 - 13:31 trong Số học

 Với mỗi số nguyên dương $k$, ta định nghĩa $S(k)$ là tổng các ước nguyên tố phân biệt của $k$. Ví dụ $S(20)=7,S(2)=2,S(1)=0$. Tìm tất cả các số nguyên dương $n$ để $S(2^n+1)=S(n)$




#735043 TOPIC [MỘT SỐ BÀI TOÁN SỐ HỌC]

Đã gửi bởi KietLW9 on 21-09-2022 - 14:23 trong Số học

$\textbf{Bài toán 6.}$ Xét $p$ là số nguyên tố có dạng $4k+1$ trong đó $k$ là số nguyên dương thỏa mãn $p^2|2^{p-1}-1$. Gọi $q$ là ước nguyên tố lớn nhất của $2^p-1$ Chứng minh rằng: $2^q > (6p)^p$

 

$\textbf{Lời giải.}$ 

Ta có phân tích tiêu chuẩn của $2^p-1$ như sau: $2^p-1=q_{1}^{a_1}q_{2}^{a_2}...q_{k}^{a_k}$ trong đó $q_1,q_2,...,q_k$ là các ước nguyên tố lẻ phân biệt của $2^p-1$.

Đầu tiên ta sẽ chứng minh $q_i\equiv 1(\text{mod p}),\forall i=\overline{1,k}$

Thật vậy, ta có: $q_i|2^{q_i-1}-1$ nên $\left\{\begin{matrix}ord_{p_i}(2)|p & \\ ord_{p_i}(2)|q_i-1 & \end{matrix}\right.$ 

Từ đây dễ dàng suy ra $ord_{p_i}(2)=p$ nên ta có điều phải chứng minh

Đặt $q_i=m_ip+1$ trong đó $m_i \in \mathbb{Z}^+,\forall i =\overline{1,k}$

Kết hợp với $p^2|2^p-2$ ta suy ra $\prod_{i=1}^{k}(m_ip+1)^{a_i}\equiv 1$ (mod $p^2$). Mà $(m_ip+1)^{a_i}\equiv a_im_ip+1$ (mod $p^2$) nên $\prod_{i=1}^{k}(1+a_1m_ip)\equiv 1$ (mod $p^2$) nên $ \sum_{i=1}^{k}a_im_i\equiv 0$ (mod $p$)

Mà lại có $q_i|2^p-1\Rightarrow (\frac{2}{q_i})=1$ nên $q_i\equiv 1,7(\text{mod 8})$ do đó $8|m_i$ hoặc $6|m_ip$ nên $m_i\geq 6$

Xét $q$ là ước nguyên tố lớn nhất của $2^p-1$ sao cho $q=mp+1$ thì rõ ràng $m$ là chỉ số lớn nhất trong các số $m_i$ khi đó $2^q>2^{pm}>(2^p-1)^m>(6p)^{(a_1+a_2+...+a_k)m}>(6p)^{\sum_{i=1}^{k}a_im_i}\geq (6p)^p$

Vậy ta có điều phải chứng minh




#734970 TOPIC [MỘT SỐ BÀI TOÁN SỐ HỌC]

Đã gửi bởi KietLW9 on 15-09-2022 - 23:24 trong Số học

$\textbf{Bài toán 5.}$ Cho $n$ là số nguyên dương lớn hơn $1$ sao cho $n|6^n+7^n$. Chứng minh rằng $13|n$

$\textbf{Lời giải.}$ Rõ ràng $(n,42)=1$. Gọi $p$ là ước nguyên tố nhỏ nhất của $n$

Vì $(p,7)=1$ nên tồn tại số nguyên $p'$ sao cho $7p'\equiv 1(\text{mod p})$

Ta có: $6^n\equiv -7^n(\text{mod p})\Rightarrow (6p')^n\equiv -(7p')^n\equiv -1(\text{mod p})\Rightarrow (6p')^{2n}\equiv 1(\text{mod p})$

Gọi $h=\text{ord}_p(6p')$ thì $h|p-1$ và $h|2n$

Nếu $h$ lẻ thì $h<p$ và $h|n$ nên $h=1$

Nếu $h$ chẵn thì $h=2l$ suy ra $l<p$ và $l|n$ do đó $l=1$ và $h=2$

Vậy ta đều có $(6p')^2\equiv 1(\text{mod p})\Rightarrow (6^2-7^2)p'^2\equiv 0(\text{mod p})$

Mà rõ ràng $(p',p)=1$ do đó $p=13$

Vậy $13|n$




#734961 đối xứng của $G$ qua $EF$ nằm trên $OI$

Đã gửi bởi KietLW9 on 15-09-2022 - 08:51 trong Hình học

$\textbf{Bài toán (Phát hiện?).}$ Cho $\Delta ABC$ nhọn, không cân có $(I)$ là tâm đường tròn nội tiếp. $(I)$ tiếp xúc với $BC,CA,AB$ tại $D,E,F$. $D'$ là điểm đối xứng với $D$ qua $EF$ và $R$ là trực tâm của tam giác $BIC$. Gọi $J$ là trực tâm của $\Delta AEF$. $G$ là điểm trên $ID$ sao $JG//D'R$. Chứng minh rằng đối xứng của $G$ qua $EF$ nằm trên $OI$




#734941 Bóng đá mùa giải 2022-2023

Đã gửi bởi KietLW9 on 14-09-2022 - 14:26 trong Câu lạc bộ hâm mộ

Bayern thực sự quá mạnh đúng không ạ  :icon6:




#734940 $AT$ là trục đẳng phương của hai đường tròn $(AMC)$ và...

Đã gửi bởi KietLW9 on 14-09-2022 - 14:20 trong Hình học

$\textbf{Bài toán (Sáng tác?).}$ Cho $\Delta ABC$ nhọn, không cân có $(I)$ là tâm đường tròn nội tiếp và $H,K$ là tâm đường tròn bàng tiếp góc $B,C$ của $\Delta ABC$. $(I)$ tiếp xúc với $CA,AB$ tại $E,F$. 

a) Chứng minh rằng $(HAF)$ và $(KAE)$ cắt nhau tại một điểm $T$ khác $A$ trên đường tròn $(ABC)$

b) Gọi $M,N$ là giao điểm thứ hai của $HE,KF$ với $(I)$. Chứng minh rằng $AT$ là trục đẳng phương của hai đường tròn $(AMC)$ và $(ABN)$.




#734867 Bóng đá mùa giải 2022-2023

Đã gửi bởi KietLW9 on 08-09-2022 - 09:38 trong Câu lạc bộ hâm mộ

Bayern Munich 2 - 0 Inter Milan

Barcelona 5 -1 Plzen

Sane cú đúp, lewandowski hattrick 

chú ý thêm lewy là cấu thủ duy nhất và đầu tiên ghi hattrick cho 3 clb khác nhau ở c1